LSAT and Law School Admissions Forum

Get expert LSAT preparation and law school admissions advice from PowerScore Test Preparation.

 Administrator
PowerScore Staff
  • PowerScore Staff
  • Posts: 8917
  • Joined: Feb 02, 2011
|
#60930
Complete Question Explanation

Justify the Conclusion. The correct answer choice is (B)

When reading this stimulus, one notices a rather jarring gap between the premise and the conclusion. The premise may be diagrammed as:
  • CO = things that are commonplace and ordinary
    COA = catch our attention

    ..... ..... ..... CO :arrow: COA
Here is the conclusion:
  • MN = miracles of nature

    ..... ..... ..... COA :some: MN
This statement introduces a third variable (miracles of nature) and determines its relationship to one of the previous variables. But how did the stimulus get from “everything that is commonplace and ordinary” to “miracles of nature”? In order to prove this conclusion, the proper relationship between miracles of nature and things that are commonplace and ordinary must be established. Only answer choices (A) and (B) contain the needed elements (a quick scan for the new term “miracles of nature” immediately eliminates answer choices (C), (D), and (E)), so they are the only possible Contenders for this question.

Answer choice (A): This can be diagrammed as follows:
  • ..... ..... ..... CO :arrow: MN
Combining this answer choice with the contrapositive of the premise gives:
  • ..... ..... ..... COA :arrow: CO :arrow: MN, therefore: COA :arrow: MN
In other words, if something catches our attention then it is a miracle of nature. This is not the same inference as the conclusion given in the stimulus (there are things that fail to catch our attention but that are miracles of nature) and, since this answer choice does not logically lead to the conclusion, answer choice (A) is incorrect.

Answer choice (B): This is the correct answer choice. Answer choice (B) can be diagrammed as follows:
  • ..... ..... ..... CO :some: MN
Since “some” statements are reversible, this statement can be reordered and combined with the premise from the stimulus thus:
  • ..... ..... ..... MN :some: CO :arrow: COA
When combining relationships of different strengths, the relationship is reduced to the strength of the weakest element. In this case, the relationship becomes “Some miracles of nature will fail to catch our attention,” which is logically equivalent to the conclusion. Therefore this answer choice leads to the correct conclusion and is proven correct.

Of course, this problem can also be solved without diagramming it. Assuming this answer choice is true, any ordinary or commonplace miracles must fail to catch our attention. This assumption completes the proper relationship between all the elements of the stimulus and is therefore correct.

Answer choice (C): If all things that are commonplace and ordinary fail to catch our attention, it is inherently true that some ordinary and commonplace things will likewise fail to catch our attention. This is simply an inherent inference of the argument’s premise and does not lead to the conclusion. Further, this answer choice does not provide an explanation for the new term introduced in the conclusion and cannot be correct.

Answer choice (D): Here is the diagram of this statement:

..... ..... ..... COA :arrow: CO

This is clearly a Mistaken Reversal of the first premise. Although it is useful practice to identify this answer choice as a Mistaken Reversal, the answer is not incorrect simply because it does not logically follow from the premise. Rather, it is incorrect because it does not explain anything about “miracles of nature.” Even when this inference is added to the stimulus, it does not lead to the conclusion.

Answer choice (E): Like answer choice (D), this answer is also an invalid inference. Since “extraordinary or unusual things” are not commonplace or ordinary, this statement can be diagrammed thus:
  • ..... ..... ..... CO :arrow: COA
Once again, however, demonstrating that this answer is a Mistaken Negation is not sufficient to prove that it is incorrect. Only by proving that this statement does not bridge the gap between the premise and the conclusion can it be safely eliminated from consideration. Since this statement does not clarify the relationship between miracles of nature and things which fail to catch our attention, it is not the correct answer.
 jonwg5121
  • Posts: 38
  • Joined: Jun 06, 2015
|
#19677
Can someone please explain the question, diagram it out, and explain why answer choice (B) is correct? Thank you!
User avatar
 Dave Killoran
PowerScore Staff
  • PowerScore Staff
  • Posts: 5852
  • Joined: Mar 25, 2011
|
#19680
Hi Jon,

Thanks for the question!

The first part of the sentence is a premise, and would be diagrammed as:

  • Premise

    Commonplace
    + ..... ..... :arrow: Fail to catch attention
    Ordinary
The second part of that sentence is the conclusion:

  • Conclusion

    Fail to catch attentionMiracles of nature
I've diagrammed this for ease of understanding, but I wouldn't diagram this during the test—the relationship is straightforward enough that I don't feel like the diagram changes or enhances my understanding.

The question stem is a Justify the Conclusion question, and as I was reading I could see there was a big leap between the single premise and the conclusion. Using the Mechanistic Approach, "Fail to catch attention" is common to both the premise and conclusion, leaving "commonplace and ordinary" and "miracles of nature" of nature as the elements I want to see in the correct answer.

Answer choice (B) is the only answer to connect those two elements perfectly ((A) has "fails" in it, which isn't needed, as well as "only"). But, for some people, the word "some" in (B) throws them off. Is it a problem? No, because the conclusion doesn't say "every" or all," just that "there are things..." That's the same as "some," so we're good at that point. Inserting (B) into the argument leaves you with the following:

  • Premise

    Commonplace
    + ..... ..... :arrow: Fail to catch attention
    Ordinary


    Premise

    Commonplace
    + .....
    Ordinary ..... Miracles of nature


    Conclusion
    Fail to catch attentionMiracles of nature

So, by applying the Mechanistic approach, you should have immediately been down to (A) and (B), and then there are issues in (A) that knock it out. (B) matches perfectly and is thus correct.

Remember, if you are uncertain of whether your chosen answer in a Justify question is correct, you can re-insert it into the argument, combine it with the existing premise(s), and see if that combination produces the conclusion. If it does, it is the correct answer.

Please let me know if that helps. Thanks!
 jonwg5121
  • Posts: 38
  • Joined: Jun 06, 2015
|
#19682
Yes it does. Thanks!
 jcough346
  • Posts: 35
  • Joined: Aug 05, 2016
|
#30154
What is the mechanistic approach?
 Adam Tyson
PowerScore Staff
  • PowerScore Staff
  • Posts: 5153
  • Joined: Apr 14, 2011
|
#30170
Thanks for asking, jcough! We discuss the so-called "mechanistic approach" to Justify the Conclusion questions in our full-length course in Lesson 4, and in the most recent edition of the Logical Reasoning Bible in Chapter 10. If you have either of these resources, give a look and see what we are talking about there.

The short version is that many Justify questions do not require that you actually understand anything about what the stimulus is saying. Sometimes, all you need to do is take note of whatever the conclusion mentions that was not mentioned elsewhere in the argument. We sometimes call that the "rogue" or "new" element in the argument, and the correct answer absolutely must address that rogue element.

You also need to take note of anything in the premises that was not mentioned in the conclusion, and that, too, is a "rogue" element. The correct answer will tie a rogue element in the premises to the rogue element in the conclusion, and will usually make no mention of anything that could be found in both the premises and the conclusion.

Allow me to provide an example using nonsense:

Premise: All borbles are flammable

Conclusion: This qwerty is flammable

"Flammable" is common to both the premise and the conclusion. I am not interested in any answer that talks about flammability.

"This qwerty" is rogue in the conclusion, while "borbles" is rogue in the premise. I need to link them up in my Justify answer. How do I prove that this qwerty is flammable? If I add "this qwerty is a borble" to the argument, I have a winner.

We didn't need to understand the argument. No need to know what a borble or a qwerty is, or even what flammable means. I don't need to wonder why all borbles are flammable, or whether there are other kinds of borbles besides qwertys. All I have to do is, mechanistically, link together the rogue elements, and I have justified my conclusion.

That's it! Give it a try on your next Justify question and see if it doesn't lead you right to the correct answer, easily eliminating the losers along the way.

Get the most out of your LSAT Prep Plus subscription.

Analyze and track your performance with our Testing and Analytics Package.